Administración     

Olimpiadas de Matemáticas
Página de preparación y problemas

Selector
La base de datos contiene 1154 problemas y 775 soluciones.
OME Local
OME Nacional
OIM
OME Andalucía
Retos UJA
—20
Problema 838
Halla todas las ternas $(x,y,z)$ de números reales que son solución de la ecuación \[\sqrt{3^x(5^y+7^z)}+\sqrt{5^y(7^z+3^x)}+\sqrt{7^z(3^x+5^y)}=\sqrt{2}(3^x+5^y+7^z).\]
pistasolución 1info
Pista. Demuestra que la desigualdad $\leq$ es cierta para todo $x,y,z\in\mathbb{R}$ mediante la desigualdad de Cauchy-Schwarz.
Solución. Si aplicamos la desigualdad de Cauchy-Schwarz a los vectores \[u=\left(\sqrt{3^x},\sqrt{5^y},\sqrt{7^z}\right),\qquad v=\left(\sqrt{5^y+7^z},\sqrt{3^x+7^z},\sqrt{3^x+5^y}\right),\] obtenemos que \begin{align*} \sqrt{3^x(5^y+7^z)}+\sqrt{5^y(7^z+3^x)}+\sqrt{7^z(3^x+5^y)}&\leq\sqrt{3^x+5^y+7^z}\sqrt{(5^y+7^z)+(3^x+7^z)+(3^x+5^y)}\\ &=\sqrt{2}(3^x+5^y+7^z), \end{align*} donde hemos usado también que las exponenciales $3^x,5^7,7^z$ son números positivos. Esto nos dice que las soluciones de la ecuación son precisamente los valores que hacen de la desigualdad de Cauchy-Schwarz una igualdad. Esto equivale a que los vectores $u$ y $v$ sean proporcionales. Como están formados por números positivos, estamos buscando los $x,y,z$ tales que existe $\lambda\gt 0$ tal que \[\sqrt{3^x}=\lambda\sqrt{5^y+7^z},\qquad \sqrt{5^y}=\lambda\sqrt{3^x+7^z},\qquad \sqrt{7^z}=\lambda\sqrt{3^x+5^y}.\] Elevando al cuadrado y sumando los resultados, llegamos a que $3^x+5^y+7^z=2\lambda^2(3^x+5^y+7^z)$, luego debe ser $\lambda=\frac{1}{\sqrt{2}}$ ya que podemos cancelar $3^x+5^y+7^z\neq 0$ (recordemos que $\lambda$ es positivo). Por lo tanto, el sistema anterior nos queda \[3^x=\frac{5^y+7^z}{2},\qquad 5^y=\frac{3^x+7^z}{2},\qquad 7^z=\frac{3^x+5^y}{2}.\] Este es un sistema lineal en las incógnitas $3^x,5^y,7^z$, que es compatible indeterminado y sus soluciones son los números que verifican $3^x=5^y=7^z$. Tomando logaritmos, podemos reescribir esto como $x\log(3)=y\log(5)=z\log(7)$, luego las soluciones que buscamos pueden parametrizarse en términos de un parámetro real $a\in\mathbb{R}$ como \[(x,y,z)=\left(\frac{a}{\log(3)},\frac{a}{\log(5)},\frac{a}{\log(7)}\right).\]
Si crees que el enunciado contiene un error o imprecisión o bien crees que la información sobre la procedencia del problema es incorrecta, puedes notificarlo usando los siguientes botones:
Informar de error en enunciado Informar de procedencia del problema
Problema 832
Sean $a,b,c$ tres números reales positivos tales que $a+b+c=1$. Demuestra que \[a^{a^2+2ca}b^{b^2+2ab}c^{c^2+2bc}\geq\tfrac{1}{3}.\]
pistasolución 1info
Pista. Aplica la desigualdad entre las medias aritmética y geométrica con pesos (que es un caso particular de la desigualdad de Jensen).
Solución. La desigualdad se puede escribir de forma equivalente como \[\left(\frac{1}{a}\right)^{a^2+2ac}\left(\frac{1}{b}\right)^{b^2+2ab}\left(\frac{1}{c}\right)^{c^2+2bc}\leq 3.\] La desigualdad entre las medias aritmética y geométrica con pesos nos dice que si $x_1,x_2,x_3,w_1,w_2,w_3\geq 0$ son tales que $w_1+w_2+w_3=1$, entonces $x_1^{w_1}x_2^{w_2}x_3^{w_3}\leq w_1x_1+w_2x_2+w_3x_3$. En nuestro caso, tomaremos $x_1=\frac{1}{a}$, $x_2=\frac{1}{b}$, $x_3=\frac{1}{c}$, $w_1=a^2+2ca$, $w_2=b^2+2ab$ y $w_3^2=c^2+2bc$, que claramente verifican $w_1+w_2+w_3=(a+b+c)^2=1$. Tenemos entonces que \begin{align*} \left(\frac{1}{a}\right)^{a^2+2ac}\left(\frac{1}{b}\right)^{b^2+2ab}\left(\frac{1}{c}\right)^{c^2+2bc}&\leq \frac{a^2+2ca}{a}+\frac{b^2+2ab}{b}+\frac{c^2+2bc}{c}\\ &=3(a+b+c)=3. \end{align*}

Nota. La desigualdad entre las medias aritmética y geométrica con pesos se puede ver también como la desigualdad de Jensen para la función cóncava $f(t)=\ln(t)$. La pista la da el hecho de que los exponentes sumen $1$ pero puede ser difícil darse cuenta de que hay que invertir primero para que los signos de la desigualdad vayan en el sentido correcto.

Como los pesos son todos positivos, la igualdad se alcanza sólo cuando $x_1=x_2=x_3$, es decir, cuando $a=b=c$.

Si crees que el enunciado contiene un error o imprecisión o bien crees que la información sobre la procedencia del problema es incorrecta, puedes notificarlo usando los siguientes botones:
Informar de error en enunciado Informar de procedencia del problema
Problema 744
Sean $a,b,c$ números reales y consideremos las funciones \[f(x)=ax^2+bx+c,\qquad g(x)=cx^2+bx+a.\] Si $|f(-1)|\leq 1$, $|f(0)|\leq 1$ y $|f(1)|\leq 1$, demostrar que, para todo $x\in[-1,1]$, se cumple que \[|f(x)|\leq\tfrac{5}{4}\qquad \text{y}\qquad |g(x)|\leq 2.\]
pistasolución 1info
Pista. Mueve uno de los valores de $f(-1),f(0),f(1)$ fijando los otros dos para ver cómo se mueve $f(x)$ y llegar así a los posibles polinomios en los que se maximice el valor absoluto. En cuanto a $g(x)$, observa que es igual a $x^2f(\frac{1}{x})$.
Solución. Podemos suponer que $a\geq 0$ cambiando $f(x)$ de signo y también que $b\geq 0$ cambiando $x$ de signo (si fuera necesario). Como tenemos valores absolutos y simetría en los valores $0$ y $\pm 1$, esto no afecta al problema. Entonces, $f(x)$ es una parábola que tiene su vértice en los reales negativos (o también podría ser una recta si $a=0$, pero en ese caso se tiene claramente $|f(x)|\leq 1\lt\frac{5}{4}$). Vamos a usar ahora que si prefijamos los valores de $f(-1)$, $f(0)$ y $f(1)$ arbitrariamente, la parábola $f(x)$ está unívocamente determinada. Se cumple entonces que $f(x)$ decrece para todo $x\in (-1,0)$ en cualquiera de las siguientes situaciones:
  • Cuando decrece $f(0)$ mientras se fijan $f(-1)$ y $f(1)$.
  • Cuando crece $f(1)$ mientras se fijan $f(-1)$ y $f(0)$.
  • Cuando decrece $f(-1)$ mientras se fijan $f(0)$ y $f(1)$.

Por tanto, la parábola que toma los menores valores de $(-1,0)$ es aquella que cumple $f(-1)=f(0)=-1$ (mínimo posible) y $f(1)=1$ (máximo posible). Se obtiene fácilmente que esta parábola es $f(x)=x^2+x-1=(x+\frac{1}{2})^2-\frac{5}{4}$, lo que prueba que $f(x)\geq\frac{-5}{4}$ en $[-1,1]$ y hemos respondido así a la primera pregunta.

En cuanto a $g(x)$, comenzamos observando que \[g(x)=x^2(\tfrac{a}{x^2}+\tfrac{b}{x}+c)=x^2f(\tfrac{1}{x}).\] Por lo tanto, para acotar superiormente $|g(x)|$ en $[-1,1]$, tendremos que acotar inferiormente $|f(x)|$ en $(-\infty,-1]\cup[1,+\infty)$. Podemos seguir suponiendo que $a,b\geq 0$, luego $f(x)\geq f(-x)$ para $x\geq 1$ y solo debemos fijarnos en el caso $x\geq 1$. Un razonamiento similar al descrito más arriba nos dice que $f(x)$ crece para $x\in [1,+\infty)$ en cualquiera de las siguientes situaciones:

  • Cuando decrece $f(0)$ mientras se fijan $f(-1)$ y $f(1)$.
  • Cuando crece $f(1)$ mientras se fijan $f(-1)$ y $f(0)$.
  • Cuando crece $f(-1)$ mientras se fijan $f(0)$ y $f(1)$.
Por tanto, la parábola que tiene mayor valor posible debe cumplir que $f(-1)=f(-1)=1$ (máximo posible) y $f(0)=-1$ (mínimo posible). Esto nos da el caso óptimo $f(x)=x^2-2$. En resumen, para $x\in[-1,1]$, se tiene que \[|g(x)|=x^2|f(\tfrac{1}{x})|\leq x^2f(\tfrac{1}{|x|})\leq x^2(2\tfrac{1}{|x|^2}-1)=2-x^2\leq 2.\]

Nota. La igualdad $|f(x)|=\frac{5}{4}$ sólo se alcanza en los siguientes casos: para los polinomios $f(x)=x^2+x-1$ y $f(x)=-x^2-x+1$ en $x=\frac{-1}{2}$ y para los polinomios $f(x)=x^2-x+1$ y $f(x)=-x^2+x-1$ en $x=\frac{1}{2}$. La igualdad $|g(x)|=2$ se alcanza sólo para los polinomios $g(x)=x^2-2$ y $g(x)=2-x^2$ en $x=0$.

Si crees que el enunciado contiene un error o imprecisión o bien crees que la información sobre la procedencia del problema es incorrecta, puedes notificarlo usando los siguientes botones:
Informar de error en enunciado Informar de procedencia del problema
Problema 721
Sean $x,y,z$ números reales positivos.
  1. Si $x+y+z\geq 3$, ¿se verifica necesariamente que $\frac{1}{x}+\frac{1}{y}+\frac{1}{z}\leq 3$?
  2. Si $x+y+z\leq 3$, ¿se verifica necesariamente que $\frac{1}{x}+\frac{1}{y}+\frac{1}{z}\geq 3$?
pistasolución 1info
Pista. Utiliza la desigualdad entre las medias aritmética y armónica.
Solución. La respuesta al apartado (a) es negativa. Por ejemplo, los números $x=1$, $y=\frac{1}{2}$ y $z=\frac{3}{2}$ cumplen $x+y+z=3$, pero $\frac{1}{x}+\frac{1}{y}+\frac{1}{z}=\frac{11}{3}\gt 3$.

La respuesta al apartado (b) es afirmativa. Para verlo, usamos la desigualdad entre las medias aritmética y armónica aplicada a los tres números positivos $x,y,z$: \[\frac{3}{\frac{1}{x}+\frac{1}{y}+\frac{1}{z}}\leq \frac{x+y+z}{3}\leq 1\ \Longleftrightarrow\ \frac{1}{x}+\frac{1}{y}+\frac{1}{z}\geq 3.\]

Si crees que el enunciado contiene un error o imprecisión o bien crees que la información sobre la procedencia del problema es incorrecta, puedes notificarlo usando los siguientes botones:
Informar de error en enunciado Informar de procedencia del problema
Problema 641
Sean $x,y\geq 0$ números reales verificando $x + y = 2$. Demuestra que \[x^2y^2(x^2+y^2)\leq 2.\]
pistasolución 1info
Pista. Sustituye $x=1+t$ e $y=1-t$, siendo $0\leq t\leq 1$.
Solución. Sustituyendo $x=1+t$ e $y=1-t$ para $t\in[0,1]$, tenemos que \begin{align*}x^2y^2(x^2+y^2)&=(1+t)^2(1-t)^2((1+t)^2+(1-t)^2)\\ &=2(1-t^2)^2(1+t^2)=2(1-t^2)(1-t^4)\leq 2.\end{align*}

Nota. La igualdad se alcanza si y solo si $t=0$, es decir, cuando $x=y=1$.

Si crees que el enunciado contiene un error o imprecisión o bien crees que la información sobre la procedencia del problema es incorrecta, puedes notificarlo usando los siguientes botones:
Informar de error en enunciado Informar de procedencia del problema
José Miguel Manzano © 2010-2024. Esta página ha sido creada mediante software libre